23
$\begingroup$

I can show that the following limit exists but I am having difficulties to find it. It is $$\lim_{n\to \infty} \sum_{k=1}^n \frac{k^n}{n^n}$$ Can someone please help me?

  • 0
    How could you show that the limit exist?2012-06-28
  • 1
    @Siminore: The sum is very similar to $\int_{0}^{1}x^xdx$. Isn't it?2012-06-28
  • 0
    Numerically, for $n=1000$, I get 1.58098.2012-06-28
  • 0
    @Siminore: That happens to be only a little shy of the correct answer, $\dfrac{1}{1 - 1/e}$.2012-06-28
  • 0
    Probably. But the result is not $\int_0^1 x^x\, dx$.2012-06-28

5 Answers 5

27

An asymptotic expansion can be obtained as below. More terms can be included by using more terms in the expansions of $\exp$ and $\log$. $$ \begin{align} \sum_{k=0}^n\frac{k^n}{n^n} &=\sum_{k=0}^n\left(1-\frac{k}{n}\right)^n\\ &=\sum_{k=0}^n\exp\left(n\log\left(1-\frac{k}{n}\right)\right)\\ &=\sum_{k=0}^{\sqrt{n}}\exp\left(n\log\left(1-\frac{k}{n}\right)\right)+O\left(ne^{-\sqrt{n}}\right)\\ &=\sum_{k=0}^{\sqrt{n}}\exp\left(-k-\frac{1}{2n}k^2+O\left(\frac{k^3}{n^2}\right)\right)+O\left(ne^{-\sqrt{n}}\right)\\ &=\sum_{k=0}^{\sqrt{n}}e^{-k}\exp\left(-\frac{1}{2n}k^2+O\left(\frac{k^3}{n^2}\right)\right)+O\left(ne^{-\sqrt{n}}\right)\\ &=\sum_{k=0}^{\sqrt{n}}e^{-k}\left(1-\frac{1}{2n}k^2+O\left(\frac{k^ 4}{n^2}\right)\right)+O\left(ne^{-\sqrt{n}}\right)\\ &=\sum_{k=0}^{\sqrt{n}}e^{-k}-\frac{1}{2n}\sum_{k=0}^{\sqrt{n}}k^2e^{-k}+O\left(\frac{1}{n^2}\right)\\ &=\frac{e}{e-1}-\frac{1}{2n}\frac{e(e+1)}{(e-1)^3}+O\left(\frac{1}{n^2}\right) \end{align} $$ Several steps use $$ \sum_{k=n}^\infty e^{-k}k^m=O(e^{-n}n^m) $$ which decays faster than any power of $n$.

  • 0
    Why $$\sum\limits_{k=0}^n e^{-k}O\left(\frac{k^4}{n^2}\right)=O\left(\frac{1}{n^2}\right)$$ ?2012-06-28
  • 0
    @Norbert: Because $$\sum_{k=0}^ne^{-k}O\left(\frac{k^4}{n^2}\right)\le\left(\sum_{k=0}^\infty e^{-k}k^4\right)O\left(\frac{1}{n^2}\right)$$ !2012-06-28
  • 0
    Thakns, I'll try to learn $O$-approach in future.2012-06-28
  • 0
    I personally think that $\ln(1-x)=-x-x^2/2+O(x^3)$ only holds for $x\prec1$, so you should break the summation into $0d(n)$, where $d(n)\prec n$. PS: $f(n)\prec g(n)\iff\lim_{n\to\infty}f(n)/g(n)=0$.2012-06-29
  • 0
    Good approximation, where $\Theta_n(t)$ is not needed.2012-06-29
  • 0
    @FrankScience: Done. I did indeed forget the tail, which dies almost exponentially.2012-06-29
  • 0
    This still isn't quite correct. You can't use a Taylor expansion of $\log(1-k/n)$, because the upper bound on $k/n$ does not go to 0 as $n\to\infty$. However if you split the tail at e.g. $\lfloor\sqrt n\rfloor$, this works.2012-06-29
  • 0
    @GenericHuman I've mentioned, see preceding comment.2012-06-29
  • 0
    @robjohn LOL. Omit the tail, just like the answer on *Concrete Mathematics*, which made me misunderstood for a long time.2012-06-29
  • 0
    @FrankScience: Yes, I hadn't really read your comment since robjohn said he had addressed the problem (plus $d(n)\prec n$ looks a lot like $d(n)$d(n)=o(n)$). – 2012-06-29
  • 0
    @GenericHuman I thought about using $o(f(n))$, but eventually I used $\prec$ because both $O$ and $o$ appear seems ambiguous.2012-06-29
  • 0
    @GenericHuman: For $0\le x\le\frac12$, $\left|\log(1-x)+x+x^2/2\right|\le x^3$, so $\log(1-x)=-x-x^2/2+O(x^3)$. The Taylor series for $\log$ is fine. I did have to adjust the upper limit of the sum for $\exp\left(-\frac{1}{2n}k^2+O\left(\frac{k^3}{n^2}\right)\right)$2012-06-29
  • 0
    I really like this answer, Rob. I published a paper on this sum several years ago (see, for example, [here](http://math.stackexchange.com/questions/244657/value-of-lim-n-to-infty-frac1n2n-cdotsn-1nnn/244663#244663), and I've kept my eye out for alternative ways of getting the limiting expression. Your answer here is one of my favorites. It does a really nice job of handling the error terms.2013-01-09
  • 0
    @MikeSpivey: Thanks! I am a big fan of the Euler-Maclaurin Sum Formula. For polynomials, it is exact (in fact, in [this answer](http://math.stackexchange.com/a/156085), I give a condition for the EMS Formula to converge).2013-01-09
16

Finally, I have suffered this proof. Consider functions $$ f_n(x)=\left(1-\frac{\lfloor x\rfloor}{n}\right)^n\chi_{[0,n+1]}(x) $$ Note that $$ \int\limits_{[0,+\infty)} f_n(x)d\mu(x)=\sum\limits_{k=0}^n\int\limits_{[k,k+1)}\left(1-\frac{\lfloor x\rfloor}{n}\right)^nd\mu(x)= \sum\limits_{k=0}^n\left(1-\frac{k}{n}\right)^n $$ $$ \lim\limits_{n\to\infty}f_n(x)=\lim\limits_{n\to\infty}\left(1-\frac{\lfloor x\rfloor}{n}\right)^n\cdot \lim\limits_{n\to\infty}\chi_{[0,n+1]}(x)=e^{\lfloor x\rfloor} $$ One may check that $\{f_n:n\in\mathbb{N}\}$ is a non-decreasing sequence of non-negative functions, then using monotone convergence theorem we get $$ \lim\limits_{n\to\infty}\sum\limits_{k=0}^n\left(\frac{k}{n}\right)^n= \lim\limits_{n\to\infty}\sum\limits_{k=0}^n\left(1-\frac{k}{n}\right)^n= \lim\limits_{n\to\infty}\int\limits_{[0,+\infty)} f_n(x)d\mu(x)= $$ $$ \int\limits_{[0,+\infty)} \lim\limits_{n\to\infty}f_n(x)d\mu(x)= \int\limits_{[0,+\infty)} e^{\lfloor x\rfloor}d\mu(x)= \sum\limits_{k=0}^\infty e^{-k}=\frac{1}{1-e^{-1}} $$

  • 1
    You must justify the interchange of the limit and the sum in the second equality. One way to do this is to note that $(1-k/m)^m<(1-k/n)^n$ if $k+1\leq m2012-06-28
  • 0
    You could just use the sequence version of the [Monotone Convergence Theorem](http://en.wikipedia.org/wiki/Monotone_convergence_theorem).2012-06-28
  • 0
    Thanks, I didn't knew that2012-06-28
  • 0
    Could you compute the asymptotics, not just the limits. I'm very interested in such theory.2012-06-28
  • 0
    @FrankScience Why don't you use [Euler–Maclaurin formula](http://en.wikipedia.org/wiki/Euler%E2%80%93Maclaurin_formula)?2012-06-28
  • 0
    @Norbert Because yesterday I thought [Poisson summation formula](http://en.wikipedia.org/wiki/Poisson_summation_formula) is more powerful in this problem than Euler-Maclaurin summation formula, where $\Theta_n(t)$ is periodic function of $t$.2012-06-29
7

Let's notice a few things. All the terms are positive, bounded between $0$ and $1$, and there is a term that is exactly $1$. What about the next largest term?

So we ask ourselves what $\lim \limits_{n \to \infty} \left( \dfrac{n-1}{n} \right)^n$ is, and after a little calculation we see that this limit is $1/e$. The 'next' term involves $\lim \limits_{n \to \infty} \left( \dfrac{n-2}{n} \right)^n = e^{-2}$. So heuristically, we would expect the limit to be

$$1 + e^{-1} + e^{-2} + \dots = \frac{1}{1-\frac{1}{e}}$$

Working only a little harder, you can justify that this is the limit.

  • 0
    I wonder if it is possible to prove that $$\lim_{n \to +\infty} \sum_{k=1}^n \left( \frac{k^n}{n^n}-e^{-k} \right)=0.$$2012-06-28
  • 0
    Sorry: the limit should be 1, in the previous comment.2012-06-28
  • 0
    @Siminore: Have you tried? It's really not bad at all, as long as you know the dominated convergence theorem and/or the monotone convergence theorem. In fact, that series is absolutely convergent, so you can do a whole lot of things to it2012-06-28
  • 0
    Yes, it's a matter of passing to the limit inside a series.2012-06-28
6

Just for reference: With aid of some fancy theorem, you can skip most of hard analysis. As in other answers, we begin by writing

$$ \sum_{k=1}^{n} \left( \frac{k}{n}\right)^n \ \overset{k \to n-k}{=} \ \sum_{k=0}^{n-1} \left( 1 - \frac{k}{n}\right)^n \ = \ \sum_{k=0}^{\infty} \left( 1 - \frac{k}{n}\right)^n \mathbf{1}_{\{k < n\}}, $$

where $\mathbf{1}_{\{k < n\}}$ is the indicator function which takes value $1$ if $k < n$ and $0$ otherwise. Now for each $0 \leq k < n$, utilizing the inequality $\log(1-x) \leq -x$ which holds for all $x \in [0,1)$ shows that

$$ \left( 1 - \frac{k}{n}\right)^n = e^{n \log(1 - \frac{k}{n})} \leq e^{-k}. $$

Since $\sum_{k=0}^{\infty} e^{-k} < \infty$, by the dominated convergence theorem we can interchange the infinite sum and the limit:

$$ \lim_{n\to\infty} \sum_{k=1}^{n} \left( \frac{k}{n}\right)^n = \sum_{k=0}^{\infty} \lim_{n\to\infty} \left( 1 - \frac{k}{n}\right)^n \mathbf{1}_{\{k < n\}} = \sum_{k=0}^{\infty} e^{-k} = \frac{1}{1 - e^{-1}}. $$

  • 1
    Ah, so *this* is what mixedmath was alluding to. Very nicely done. (+1)2017-09-27
  • 0
    Maybe this is a basic question, but how did you get the first equality?2017-09-27
  • 0
    @user372003, I added a bit of details to my answer. But basically, I replaced the index $k$ by $n-k$.2017-09-27
3

$\sum_{k=1}^n(k/n)^n=\sum_{00}b_k(n)+O(\Sigma_a(n))+O(\Sigma_b(n))+O(\Sigma_c(n))$$ where $$\sum_{k>0}b_k(n)=\sum_{k>0}e^{-k}=\frac e{e-1}$$ and \begin{align*} \Sigma_b(n)&=\sum_{k>n^{1/3}}e^{-k}=O(e^{n^{1/3}})\\ \Sigma_a(n)&=\sum_{n^{1/3}0}e^{-k}k^2/n=O\left(\frac 1n\right) \end{align*} Hence, we have $\sum_{0

Can anybody give a more accurate approximation? The key to the approximation is to find the asymptotics for $\sum_{k>0}\exp(-k-k^2/2n)$, like the Bell sum $\sum_{k>0}e^{-k^2/n}$.

Edit anon pointed out that it's theta function: $\sum_ke^{-(k+t)^2/n}$, so the Fourier series works pretty well for the asymptotics: $$\Theta_n(t)=\sqrt{\pi n}\left(1+2e^{-\pi^2 n}(\cos2\pi t)+2e^{-4\pi^2 n}(\cos4\pi t)+2e^{-9\pi^2 n}(\cos6\pi t)+\cdots\right)$$ But I have no idea about Fourier series because I know very little about calculus!

  • 0
    In [my answer](http://math.stackexchange.com/a/164269), I show how to get more accuracy by including more terms in in the series for $\log$ and $\exp$.2012-06-28
  • 0
    @robjohn Thanks.2012-06-29